K
Khách

Hãy nhập câu hỏi của bạn vào đây, nếu là tài khoản VIP, bạn sẽ được ưu tiên trả lời.

31 tháng 10 2018

\(A=\frac{ab}{a+c+b+c}+\frac{bc}{a+b+a+c}+\frac{ca}{a+b+b+c}\)

\(\le\frac{1}{4}\left(\frac{ab}{a+c}+\frac{ab}{b+c}+\frac{bc}{a+b}+\frac{bc}{a+c}+\frac{ca}{a+b}+\frac{ca}{b+c}\right)\)

\(=\frac{1}{4}\left(a+b+c\right)=\frac{1}{4}\)

Nên max A là \(\frac{1}{4}\) khi \(a=b=c=\frac{1}{3}\)

18 tháng 2 2020

Dùng bđt AM - GM cho 7 số; 2 số và 3 số không âm, ta được:

\(a^3c^2+a^3c^2+a^3c^2+b^3a^2+b^3a^2+1+1\ge7a\)(1)

\(b^3a^2+b^3a^2+b^3a^2+c^3b^2+c^3b^2+1+1\ge7b\)(2)

\(c^3b^2+c^3b^2+c^3b^2+a^3c^2+a^3c^2+1+1\ge7c\)(3)

\(\frac{a+b+c}{2}+\frac{9}{2\left(a+b+c\right)}\ge3\)

\(a+b+c\ge3\)

Từ (1); (2); (3) suy ra \(a^3c^2+b^3a^2+c^3b^2\ge\frac{7\left(a+b+c\right)}{5}-\frac{6}{5}\)

\(P=\text{Σ}_{cyc}\frac{a}{b^2}+\frac{9}{2\left(a+b+c\right)}=\text{Σ}_{cyc}a^3c^2+\frac{9}{2\left(a+b+c\right)}\)

\(\ge\frac{7\left(a+b+c\right)}{5}+\frac{9}{2\left(a+b+c\right)}-\frac{6}{5}\)

\(=\frac{a+b+c}{2}+\frac{9}{2\left(a+b+c\right)}+\frac{9\left(a+b+c\right)}{10}-\frac{6}{5}\)

\(\ge3+\frac{9}{10}.3-\frac{6}{5}=\frac{9}{2}\)

Đẳng thức xảy ra khi a = b = c = 1

tích mình đi

làm ơn

rùi mình

tích lại

thanks

27 tháng 7 2018

k mk đi 

30 tháng 6 2020

Theo đánh giá bởi Bunhiacopski ta dễ có:

\(\frac{a}{b^4+c^4+a}=\frac{a\left(1+1+a^3\right)}{\left(b^4+c^4+a\right)\left(1+1+a^3\right)}\le\frac{a^4+a+a}{\left(a^2+b^2+c^2\right)^2}\)

Tương tự rồi cộng lại ta được:

\(T\le\frac{a^4+b^4+c^4+2a+2b+2c}{\left(a^2+b^2+c^2\right)^2}\)

Ta đi chứng minh:

\(\frac{a^4+b^4+c^4+2a+2b+2c}{\left(a^2+b^2+c^2\right)^2}\le1\Leftrightarrow\left(a^2+b^2+c^2\right)^2\ge a^4+b^4+c^4+2a+2b+2c\)

\(\Leftrightarrow a^2b^2+b^2c^2+c^2a^2\ge a+b+c\)

Mà \(LHS\ge abc\left(a+b+c\right)=a+b+c\Rightarrow T\le1\)

Đẳng thức xảy ra tại a=b=c=1

26 tháng 6 2020

\(A=\frac{1}{a^2\left(b+c\right)}+\frac{1}{b^2\left(c+a\right)}+\frac{1}{c^2\left(a+b\right)}\)

\(=\frac{abc}{a^2\left(b+c\right)}+\frac{abc}{b^2\left(c+a\right)}+\frac{abc}{c^2\left(a+b\right)}\)

\(=\frac{bc}{ab+ac}+\frac{ac}{bc+ba}+\frac{ab}{ac+bc}\)

Đặt: \(ab=x;bc=y;ac=z\)=> xyz = 1; x,y,z>0

\(A=\frac{y}{x+z}+\frac{z}{y+x}+\frac{x}{z+y}=\frac{y^2}{xy+yz}+\frac{z^2}{yz+xz}+\frac{x^2}{zx+xy}\)

\(\ge\frac{\left(x+y+z\right)^2}{2\left(xy+xz+xz\right)}\ge\frac{3\left(xy+yz+zx\right)}{2\left(xy+yz+zx\right)}=\frac{3}{2}\)

Dấu "=" xảy ra <=> x = y = z= 1 => a = b = c = 1

Vậy gtnn của A = 3/2 tại  a = b = c = 1

13 tháng 9 2017

Áp dụng BĐT AM-GM ta có:

\(\frac{a^3}{a^2+b^2}=a-\frac{ab^2}{a^2+b^2}\ge a-\frac{ab^2}{2ab}=a-\frac{b}{2}\)

Tương tự cho 2 BĐT còn lại ta cũng có:

\(\frac{b^3}{b^2+c^2}\ge b-\frac{c}{2};\frac{c^3}{a^2+b^2}\ge c-\frac{a}{2}\)

Cộng theo vế 3 BĐT trên ta có: 

\(P\ge\left(a+b+c\right)-\frac{a+b+c}{2}\ge3-\frac{3}{2}=\frac{3}{2}\)

Khi \(a=b=c=1\)

12 tháng 9 2017

 ta biến đổi a^2/(a+b^2)=a^3/a(a+b^2) áp dụng bất đẳng thức cosi cho 3 số a^3/a(a+b^2) ,a/2,a+b^2 
ta đc a^3/a(a+b^2)+a/2+(a+b^2)/a>= 3a/2 tương tự b^3/b(b+c^2)+b/2+(b+c^2)/4>=3b/2 
c^3/c(c+a^2)+c/2+(c+a^2)/4>=3c/2 
đặt biểu thức đầu là P Ta có P +(a+b+c)/2+(a+b+c+a^2+b^2+c^2)/4>=3/2(a+... 
mặt khác (a+b+c)^2=<3(a^2+b^2+c^2) => a^2+b^2+c^2>=3 
thay vào =>P>=3/2 DẤU "=" XẢY RA <=> A=B=C=1 
CHÚC BẠN THÀNH CÔNG